A formula for delta function in quantum mechanicsFactor 2 in Heisenberg Uncertainty Principle: Which formula is correct?Practical way of expressing the $delta$-functionHow is the Dirac delta function used in classical mechanics?The contradiction between Gell-mann Low theorem and the identity of Møller operator $HOmega_+=Omega_+H_0$Normalization condition of infinite basisWigner function of position eigenket?Triple Delta Potential in Quantum MechanicsDerivative of delta functionDelta function from poles of Green's functionA Naive Question about Delta Function and Wick Rotation

How to pronounce "I ♥ Huckabees"?

Most cost effective thermostat setting: consistent temperature vs. lowest temperature possible

Does this sum go infinity?

Are ETF trackers fundamentally better than individual stocks?

Do the common programs (for example: "ls", "cat") in Linux and BSD come from the same source code?

Custom alignment for GeoMarkers

Is there a symmetric-key algorithm which we can use for creating a signature?

How should I state my peer review experience in the CV?

ERC721: How to get the owned tokens of an address

World War I as a war of liberals against authoritarians?

What's the meaning of a knight fighting a snail in medieval book illustrations?

Is there a place to find the pricing for things not mentioned in the PHB? (non-magical)

Adventure Game (text based) in C++

How could a scammer know the apps on my phone / iTunes account?

PTIJ: Who should I vote for? (21st Knesset Edition)

Is honey really a supersaturated solution? Does heating to un-crystalize redissolve it or melt it?

Is a party consisting of only a bard, a cleric, and a warlock functional long-term?

Did Ender ever learn that he killed Stilson and/or Bonzo?

Why did it take so long to abandon sail after steamships were demonstrated?

Instead of a Universal Basic Income program, why not implement a "Universal Basic Needs" program?

As a new Ubuntu desktop 18.04 LTS user, do I need to use ufw for a firewall or is iptables sufficient?

Why one should not leave fingerprints on bulbs and plugs?

Are Roman Catholic priests ever addressed as pastor

Why Choose Less Effective Armour Types?



A formula for delta function in quantum mechanics


Factor 2 in Heisenberg Uncertainty Principle: Which formula is correct?Practical way of expressing the $delta$-functionHow is the Dirac delta function used in classical mechanics?The contradiction between Gell-mann Low theorem and the identity of Møller operator $HOmega_+=Omega_+H_0$Normalization condition of infinite basisWigner function of position eigenket?Triple Delta Potential in Quantum MechanicsDerivative of delta functionDelta function from poles of Green's functionA Naive Question about Delta Function and Wick Rotation













2












$begingroup$


I met a formula for delta function in a QM book ( not in English). The formula is used in the scattering theory. Its form is
$$lim_alpharightarrow inftyexp[ialpha x]=2idelta(x), ~~(xgeq 0).$$ Before, I never saw this form. Is this formula correct? Where can I find relevant references?










share|cite|improve this question









$endgroup$







  • 1




    $begingroup$
    Which non-English QM book?
    $endgroup$
    – Qmechanic
    12 hours ago











  • $begingroup$
    @Qmechanic The QM book is not written in English.
    $endgroup$
    – user10709
    12 hours ago










  • $begingroup$
    Even as a teacher of QM for 25 years I never saw this formula. I must check if it does make sense on terms of distributions...
    $endgroup$
    – Jhor
    11 hours ago










  • $begingroup$
    I don't think it can be an alternative definition of the Delta function. The Delta function needs to at least span in a neighborhood of $x=0$ for the integration properties of the Delta function to have a meaning. Note that I am not implying that the above relationship is wrong; it could be satisfied for $x>0$, but it won't serve as a definition of $delta (X)$.
    $endgroup$
    – Daniel Duque
    8 hours ago






  • 1




    $begingroup$
    @user10709 Which QM book? (That's what Qmechanic asked).
    $endgroup$
    – Alex Strasser
    7 hours ago















2












$begingroup$


I met a formula for delta function in a QM book ( not in English). The formula is used in the scattering theory. Its form is
$$lim_alpharightarrow inftyexp[ialpha x]=2idelta(x), ~~(xgeq 0).$$ Before, I never saw this form. Is this formula correct? Where can I find relevant references?










share|cite|improve this question









$endgroup$







  • 1




    $begingroup$
    Which non-English QM book?
    $endgroup$
    – Qmechanic
    12 hours ago











  • $begingroup$
    @Qmechanic The QM book is not written in English.
    $endgroup$
    – user10709
    12 hours ago










  • $begingroup$
    Even as a teacher of QM for 25 years I never saw this formula. I must check if it does make sense on terms of distributions...
    $endgroup$
    – Jhor
    11 hours ago










  • $begingroup$
    I don't think it can be an alternative definition of the Delta function. The Delta function needs to at least span in a neighborhood of $x=0$ for the integration properties of the Delta function to have a meaning. Note that I am not implying that the above relationship is wrong; it could be satisfied for $x>0$, but it won't serve as a definition of $delta (X)$.
    $endgroup$
    – Daniel Duque
    8 hours ago






  • 1




    $begingroup$
    @user10709 Which QM book? (That's what Qmechanic asked).
    $endgroup$
    – Alex Strasser
    7 hours ago













2












2








2





$begingroup$


I met a formula for delta function in a QM book ( not in English). The formula is used in the scattering theory. Its form is
$$lim_alpharightarrow inftyexp[ialpha x]=2idelta(x), ~~(xgeq 0).$$ Before, I never saw this form. Is this formula correct? Where can I find relevant references?










share|cite|improve this question









$endgroup$




I met a formula for delta function in a QM book ( not in English). The formula is used in the scattering theory. Its form is
$$lim_alpharightarrow inftyexp[ialpha x]=2idelta(x), ~~(xgeq 0).$$ Before, I never saw this form. Is this formula correct? Where can I find relevant references?







quantum-mechanics dirac-delta-distributions






share|cite|improve this question













share|cite|improve this question











share|cite|improve this question




share|cite|improve this question










asked 12 hours ago









user10709user10709

562




562







  • 1




    $begingroup$
    Which non-English QM book?
    $endgroup$
    – Qmechanic
    12 hours ago











  • $begingroup$
    @Qmechanic The QM book is not written in English.
    $endgroup$
    – user10709
    12 hours ago










  • $begingroup$
    Even as a teacher of QM for 25 years I never saw this formula. I must check if it does make sense on terms of distributions...
    $endgroup$
    – Jhor
    11 hours ago










  • $begingroup$
    I don't think it can be an alternative definition of the Delta function. The Delta function needs to at least span in a neighborhood of $x=0$ for the integration properties of the Delta function to have a meaning. Note that I am not implying that the above relationship is wrong; it could be satisfied for $x>0$, but it won't serve as a definition of $delta (X)$.
    $endgroup$
    – Daniel Duque
    8 hours ago






  • 1




    $begingroup$
    @user10709 Which QM book? (That's what Qmechanic asked).
    $endgroup$
    – Alex Strasser
    7 hours ago












  • 1




    $begingroup$
    Which non-English QM book?
    $endgroup$
    – Qmechanic
    12 hours ago











  • $begingroup$
    @Qmechanic The QM book is not written in English.
    $endgroup$
    – user10709
    12 hours ago










  • $begingroup$
    Even as a teacher of QM for 25 years I never saw this formula. I must check if it does make sense on terms of distributions...
    $endgroup$
    – Jhor
    11 hours ago










  • $begingroup$
    I don't think it can be an alternative definition of the Delta function. The Delta function needs to at least span in a neighborhood of $x=0$ for the integration properties of the Delta function to have a meaning. Note that I am not implying that the above relationship is wrong; it could be satisfied for $x>0$, but it won't serve as a definition of $delta (X)$.
    $endgroup$
    – Daniel Duque
    8 hours ago






  • 1




    $begingroup$
    @user10709 Which QM book? (That's what Qmechanic asked).
    $endgroup$
    – Alex Strasser
    7 hours ago







1




1




$begingroup$
Which non-English QM book?
$endgroup$
– Qmechanic
12 hours ago





$begingroup$
Which non-English QM book?
$endgroup$
– Qmechanic
12 hours ago













$begingroup$
@Qmechanic The QM book is not written in English.
$endgroup$
– user10709
12 hours ago




$begingroup$
@Qmechanic The QM book is not written in English.
$endgroup$
– user10709
12 hours ago












$begingroup$
Even as a teacher of QM for 25 years I never saw this formula. I must check if it does make sense on terms of distributions...
$endgroup$
– Jhor
11 hours ago




$begingroup$
Even as a teacher of QM for 25 years I never saw this formula. I must check if it does make sense on terms of distributions...
$endgroup$
– Jhor
11 hours ago












$begingroup$
I don't think it can be an alternative definition of the Delta function. The Delta function needs to at least span in a neighborhood of $x=0$ for the integration properties of the Delta function to have a meaning. Note that I am not implying that the above relationship is wrong; it could be satisfied for $x>0$, but it won't serve as a definition of $delta (X)$.
$endgroup$
– Daniel Duque
8 hours ago




$begingroup$
I don't think it can be an alternative definition of the Delta function. The Delta function needs to at least span in a neighborhood of $x=0$ for the integration properties of the Delta function to have a meaning. Note that I am not implying that the above relationship is wrong; it could be satisfied for $x>0$, but it won't serve as a definition of $delta (X)$.
$endgroup$
– Daniel Duque
8 hours ago




1




1




$begingroup$
@user10709 Which QM book? (That's what Qmechanic asked).
$endgroup$
– Alex Strasser
7 hours ago




$begingroup$
@user10709 Which QM book? (That's what Qmechanic asked).
$endgroup$
– Alex Strasser
7 hours ago










2 Answers
2






active

oldest

votes


















2












$begingroup$

The Fourier transform of a smooth test function $f(x)$ defined on the whole real line will be tend to zero as $alphato infty$. If, however, it is non-zero only for $xge0$ and has a jump at the origin from $0$ on the negative axis to $f(0)$, and is thereafter smooth ,then it will go to zero as $-f(0)/ialpha$. So have you missed out a factor of $alpha$ in your formula?



Something like
$$
lim_alphato infty alpha e^ialpha x= i delta(x)?
$$

I don't know about the "$2$" though....






share|cite|improve this answer









$endgroup$




















    2












    $begingroup$

    First of all, it is the book's author's responsibility to prove the formula or give a reference. Second, I suspect the formula is wrong. My reasoning (which is not conclusive) is as follows.



    As you know, delta-function is not a function, but a distribution, and for a test function $f(x)$ we have $int_-infty^infty f(x)delta(x)dx=f(0)$. Let us assume (and this may be controversial) that $int_0^infty f(x)delta(x)dx=frac12f(0)$. Then, assuming the formula is correct, we obtain $$int_0^infty f(x)frac12ilim_alphatoinftyexp(ialpha x)dx=frac12f(0).$$Let us try $f(x)=exp(-s x)$, where $s>0$. We obtain:$$int_0^infty exp(-s x)frac12ilim_alphatoinftyexp(ialpha x)dx=frac12ilim_alphatoinftyfrac-1ialpha-s=frac12.$$This does not look correct.



    EDIT (3/16/2019): @mike stone's suggestion may make the formula correct: if we consider the following initial formula: $$lim_alphatoinftyalphaexp(ialpha x)=2idelta(x)$$ for $xgeq 0$, my calculation seems to give a correct result.






    share|cite|improve this answer











    $endgroup$












      Your Answer





      StackExchange.ifUsing("editor", function ()
      return StackExchange.using("mathjaxEditing", function ()
      StackExchange.MarkdownEditor.creationCallbacks.add(function (editor, postfix)
      StackExchange.mathjaxEditing.prepareWmdForMathJax(editor, postfix, [["$", "$"], ["\\(","\\)"]]);
      );
      );
      , "mathjax-editing");

      StackExchange.ready(function()
      var channelOptions =
      tags: "".split(" "),
      id: "151"
      ;
      initTagRenderer("".split(" "), "".split(" "), channelOptions);

      StackExchange.using("externalEditor", function()
      // Have to fire editor after snippets, if snippets enabled
      if (StackExchange.settings.snippets.snippetsEnabled)
      StackExchange.using("snippets", function()
      createEditor();
      );

      else
      createEditor();

      );

      function createEditor()
      StackExchange.prepareEditor(
      heartbeatType: 'answer',
      autoActivateHeartbeat: false,
      convertImagesToLinks: false,
      noModals: true,
      showLowRepImageUploadWarning: true,
      reputationToPostImages: null,
      bindNavPrevention: true,
      postfix: "",
      imageUploader:
      brandingHtml: "Powered by u003ca class="icon-imgur-white" href="https://imgur.com/"u003eu003c/au003e",
      contentPolicyHtml: "User contributions licensed under u003ca href="https://creativecommons.org/licenses/by-sa/3.0/"u003ecc by-sa 3.0 with attribution requiredu003c/au003e u003ca href="https://stackoverflow.com/legal/content-policy"u003e(content policy)u003c/au003e",
      allowUrls: true
      ,
      noCode: true, onDemand: true,
      discardSelector: ".discard-answer"
      ,immediatelyShowMarkdownHelp:true
      );



      );













      draft saved

      draft discarded


















      StackExchange.ready(
      function ()
      StackExchange.openid.initPostLogin('.new-post-login', 'https%3a%2f%2fphysics.stackexchange.com%2fquestions%2f466780%2fa-formula-for-delta-function-in-quantum-mechanics%23new-answer', 'question_page');

      );

      Post as a guest















      Required, but never shown

























      2 Answers
      2






      active

      oldest

      votes








      2 Answers
      2






      active

      oldest

      votes









      active

      oldest

      votes






      active

      oldest

      votes









      2












      $begingroup$

      The Fourier transform of a smooth test function $f(x)$ defined on the whole real line will be tend to zero as $alphato infty$. If, however, it is non-zero only for $xge0$ and has a jump at the origin from $0$ on the negative axis to $f(0)$, and is thereafter smooth ,then it will go to zero as $-f(0)/ialpha$. So have you missed out a factor of $alpha$ in your formula?



      Something like
      $$
      lim_alphato infty alpha e^ialpha x= i delta(x)?
      $$

      I don't know about the "$2$" though....






      share|cite|improve this answer









      $endgroup$

















        2












        $begingroup$

        The Fourier transform of a smooth test function $f(x)$ defined on the whole real line will be tend to zero as $alphato infty$. If, however, it is non-zero only for $xge0$ and has a jump at the origin from $0$ on the negative axis to $f(0)$, and is thereafter smooth ,then it will go to zero as $-f(0)/ialpha$. So have you missed out a factor of $alpha$ in your formula?



        Something like
        $$
        lim_alphato infty alpha e^ialpha x= i delta(x)?
        $$

        I don't know about the "$2$" though....






        share|cite|improve this answer









        $endgroup$















          2












          2








          2





          $begingroup$

          The Fourier transform of a smooth test function $f(x)$ defined on the whole real line will be tend to zero as $alphato infty$. If, however, it is non-zero only for $xge0$ and has a jump at the origin from $0$ on the negative axis to $f(0)$, and is thereafter smooth ,then it will go to zero as $-f(0)/ialpha$. So have you missed out a factor of $alpha$ in your formula?



          Something like
          $$
          lim_alphato infty alpha e^ialpha x= i delta(x)?
          $$

          I don't know about the "$2$" though....






          share|cite|improve this answer









          $endgroup$



          The Fourier transform of a smooth test function $f(x)$ defined on the whole real line will be tend to zero as $alphato infty$. If, however, it is non-zero only for $xge0$ and has a jump at the origin from $0$ on the negative axis to $f(0)$, and is thereafter smooth ,then it will go to zero as $-f(0)/ialpha$. So have you missed out a factor of $alpha$ in your formula?



          Something like
          $$
          lim_alphato infty alpha e^ialpha x= i delta(x)?
          $$

          I don't know about the "$2$" though....







          share|cite|improve this answer












          share|cite|improve this answer



          share|cite|improve this answer










          answered 9 hours ago









          mike stonemike stone

          7,2251225




          7,2251225





















              2












              $begingroup$

              First of all, it is the book's author's responsibility to prove the formula or give a reference. Second, I suspect the formula is wrong. My reasoning (which is not conclusive) is as follows.



              As you know, delta-function is not a function, but a distribution, and for a test function $f(x)$ we have $int_-infty^infty f(x)delta(x)dx=f(0)$. Let us assume (and this may be controversial) that $int_0^infty f(x)delta(x)dx=frac12f(0)$. Then, assuming the formula is correct, we obtain $$int_0^infty f(x)frac12ilim_alphatoinftyexp(ialpha x)dx=frac12f(0).$$Let us try $f(x)=exp(-s x)$, where $s>0$. We obtain:$$int_0^infty exp(-s x)frac12ilim_alphatoinftyexp(ialpha x)dx=frac12ilim_alphatoinftyfrac-1ialpha-s=frac12.$$This does not look correct.



              EDIT (3/16/2019): @mike stone's suggestion may make the formula correct: if we consider the following initial formula: $$lim_alphatoinftyalphaexp(ialpha x)=2idelta(x)$$ for $xgeq 0$, my calculation seems to give a correct result.






              share|cite|improve this answer











              $endgroup$

















                2












                $begingroup$

                First of all, it is the book's author's responsibility to prove the formula or give a reference. Second, I suspect the formula is wrong. My reasoning (which is not conclusive) is as follows.



                As you know, delta-function is not a function, but a distribution, and for a test function $f(x)$ we have $int_-infty^infty f(x)delta(x)dx=f(0)$. Let us assume (and this may be controversial) that $int_0^infty f(x)delta(x)dx=frac12f(0)$. Then, assuming the formula is correct, we obtain $$int_0^infty f(x)frac12ilim_alphatoinftyexp(ialpha x)dx=frac12f(0).$$Let us try $f(x)=exp(-s x)$, where $s>0$. We obtain:$$int_0^infty exp(-s x)frac12ilim_alphatoinftyexp(ialpha x)dx=frac12ilim_alphatoinftyfrac-1ialpha-s=frac12.$$This does not look correct.



                EDIT (3/16/2019): @mike stone's suggestion may make the formula correct: if we consider the following initial formula: $$lim_alphatoinftyalphaexp(ialpha x)=2idelta(x)$$ for $xgeq 0$, my calculation seems to give a correct result.






                share|cite|improve this answer











                $endgroup$















                  2












                  2








                  2





                  $begingroup$

                  First of all, it is the book's author's responsibility to prove the formula or give a reference. Second, I suspect the formula is wrong. My reasoning (which is not conclusive) is as follows.



                  As you know, delta-function is not a function, but a distribution, and for a test function $f(x)$ we have $int_-infty^infty f(x)delta(x)dx=f(0)$. Let us assume (and this may be controversial) that $int_0^infty f(x)delta(x)dx=frac12f(0)$. Then, assuming the formula is correct, we obtain $$int_0^infty f(x)frac12ilim_alphatoinftyexp(ialpha x)dx=frac12f(0).$$Let us try $f(x)=exp(-s x)$, where $s>0$. We obtain:$$int_0^infty exp(-s x)frac12ilim_alphatoinftyexp(ialpha x)dx=frac12ilim_alphatoinftyfrac-1ialpha-s=frac12.$$This does not look correct.



                  EDIT (3/16/2019): @mike stone's suggestion may make the formula correct: if we consider the following initial formula: $$lim_alphatoinftyalphaexp(ialpha x)=2idelta(x)$$ for $xgeq 0$, my calculation seems to give a correct result.






                  share|cite|improve this answer











                  $endgroup$



                  First of all, it is the book's author's responsibility to prove the formula or give a reference. Second, I suspect the formula is wrong. My reasoning (which is not conclusive) is as follows.



                  As you know, delta-function is not a function, but a distribution, and for a test function $f(x)$ we have $int_-infty^infty f(x)delta(x)dx=f(0)$. Let us assume (and this may be controversial) that $int_0^infty f(x)delta(x)dx=frac12f(0)$. Then, assuming the formula is correct, we obtain $$int_0^infty f(x)frac12ilim_alphatoinftyexp(ialpha x)dx=frac12f(0).$$Let us try $f(x)=exp(-s x)$, where $s>0$. We obtain:$$int_0^infty exp(-s x)frac12ilim_alphatoinftyexp(ialpha x)dx=frac12ilim_alphatoinftyfrac-1ialpha-s=frac12.$$This does not look correct.



                  EDIT (3/16/2019): @mike stone's suggestion may make the formula correct: if we consider the following initial formula: $$lim_alphatoinftyalphaexp(ialpha x)=2idelta(x)$$ for $xgeq 0$, my calculation seems to give a correct result.







                  share|cite|improve this answer














                  share|cite|improve this answer



                  share|cite|improve this answer








                  edited 8 hours ago

























                  answered 10 hours ago









                  akhmeteliakhmeteli

                  18.4k21843




                  18.4k21843



























                      draft saved

                      draft discarded
















































                      Thanks for contributing an answer to Physics Stack Exchange!


                      • Please be sure to answer the question. Provide details and share your research!

                      But avoid


                      • Asking for help, clarification, or responding to other answers.

                      • Making statements based on opinion; back them up with references or personal experience.

                      Use MathJax to format equations. MathJax reference.


                      To learn more, see our tips on writing great answers.




                      draft saved


                      draft discarded














                      StackExchange.ready(
                      function ()
                      StackExchange.openid.initPostLogin('.new-post-login', 'https%3a%2f%2fphysics.stackexchange.com%2fquestions%2f466780%2fa-formula-for-delta-function-in-quantum-mechanics%23new-answer', 'question_page');

                      );

                      Post as a guest















                      Required, but never shown





















































                      Required, but never shown














                      Required, but never shown












                      Required, but never shown







                      Required, but never shown

































                      Required, but never shown














                      Required, but never shown












                      Required, but never shown







                      Required, but never shown







                      Popular posts from this blog

                      Oświęcim Innehåll Historia | Källor | Externa länkar | Navigeringsmeny50°2′18″N 19°13′17″Ö / 50.03833°N 19.22139°Ö / 50.03833; 19.2213950°2′18″N 19°13′17″Ö / 50.03833°N 19.22139°Ö / 50.03833; 19.221393089658Nordisk familjebok, AuschwitzInsidan tro och existensJewish Community i OświęcimAuschwitz Jewish Center: MuseumAuschwitz Jewish Center

                      Valle di Casies Indice Geografia fisica | Origini del nome | Storia | Società | Amministrazione | Sport | Note | Bibliografia | Voci correlate | Altri progetti | Collegamenti esterni | Menu di navigazione46°46′N 12°11′E / 46.766667°N 12.183333°E46.766667; 12.183333 (Valle di Casies)46°46′N 12°11′E / 46.766667°N 12.183333°E46.766667; 12.183333 (Valle di Casies)Sito istituzionaleAstat Censimento della popolazione 2011 - Determinazione della consistenza dei tre gruppi linguistici della Provincia Autonoma di Bolzano-Alto Adige - giugno 2012Numeri e fattiValle di CasiesDato IstatTabella dei gradi/giorno dei Comuni italiani raggruppati per Regione e Provincia26 agosto 1993, n. 412Heraldry of the World: GsiesStatistiche I.StatValCasies.comWikimedia CommonsWikimedia CommonsValle di CasiesSito ufficialeValle di CasiesMM14870458910042978-6

                      Typsetting diagram chases (with TikZ?) Announcing the arrival of Valued Associate #679: Cesar Manara Planned maintenance scheduled April 17/18, 2019 at 00:00UTC (8:00pm US/Eastern)How to define the default vertical distance between nodes?Draw edge on arcNumerical conditional within tikz keys?TikZ: Drawing an arc from an intersection to an intersectionDrawing rectilinear curves in Tikz, aka an Etch-a-Sketch drawingLine up nested tikz enviroments or how to get rid of themHow to place nodes in an absolute coordinate system in tikzCommutative diagram with curve connecting between nodesTikz with standalone: pinning tikz coordinates to page cmDrawing a Decision Diagram with Tikz and layout manager